Book of Proof

Home > Other > Book of Proof > Page 40
Book of Proof Page 40

by Richard Hammack


  integer f (n) is not prime.

  Proof. (Outline) Note that, because the coefficients are all positive and the

  degree is greater than 1, we have f (1) > 1. Let b = f (1) > 1. Now, the polynomial

  f (x) − b has a root 1, so f (x) − b = (x − 1)g(x) for some polynomial g. Then

  f (x) = (x − 1)g(x) + b. Now note that f (b + 1) = bg(b) + b = b(g(b) + 1). If we can

  now show that g(b) + 1 is an integer, then we have a nontrivial factoring

  f (b + 1) = b(g(b) + 1), and f (b + 1) is not prime. To complete the proof, use the

  fact that f (x) − b = (x − 1)g(x) has integer coefficients, and deduce that g(x) must

  also have integer coefficients.

  ■

  Chapter 10 Exercises

  n2 + n

  1. For every integer n ∈ N, it follows that 1 + 2 + 3 + 4 + ··· + n =

  .

  2

  Proof. We will prove this with mathematical induction.

  12 + 1

  (1) Observe that if n = 1, this statement is 1 =

  , which is obviously true.

  2

  277

  (2) Consider any integer k ≥ 1. We must show that Sk implies Sk+1. In other

  words, we must show that if 1 + 2 + 3 + 4 + · · · + k = k2+k

  2

  is true, then

  (k + 1)2 + (k + 1)

  1 + 2 + 3 + 4 + ··· + k + (k + 1) =

  2

  is also true. We use direct proof.

  Suppose k ≥ 1 and 1 + 2 + 3 + 4 + · · · + k = k2+k

  2

  . Observe that

  1 + 2 + 3 + 4 + ··· + k + (k + 1) =

  (1 + 2 + 3 + 4 + ··· + k) + (k + 1) =

  k2 + k

  k2 + k + 2(k + 1)

  + (k + 1)

  =

  2

  2

  k2 + 2k + 1 + k + 1

  =

  2

  (k + 1)2 + (k + 1)

  =

  .

  2

  Therefore we have shown that 1 + 2 + 3 + 4 + · · · + k + (k + 1) = (k+1)2+(k+1)

  2

  .

  ■

  3. For every integer n ∈ N, it follows that 13 + 23 + 33 + 43 + ··· + n3 = n2(n+1)2

  4

  .

  Proof. We will prove this with mathematical induction.

  (1) When n = 1 the statement is 13 = 12(1+1)2

  4

  = 44 = 1, which is true.

  (2) Now assume the statement is true for some integer n = k ≥ 1, that is assume

  13 + 23 + 33 + 43 + ··· + k3 = k2(k+1)2

  4

  . Observe that this implies the statement is

  true for n = k + 1.

  13 + 23 + 33 + 43 + ··· + k3 + (k + 1)3

  =

  (13 + 23 + 33 + 43 + ··· + k3) + (k + 1)3

  =

  k2(k + 1)2

  k2(k + 1)2

  4(k + 1)3

  + (k + 1)3

  =

  +

  4

  4

  4

  k2(k + 1)2 + 4(k + 1)3

  =

  4

  (k + 1)2(k2 + 4(k + 1)1)

  =

  4

  (k + 1)2(k2 + 4k + 4)

  =

  4

  (k + 1)2(k + 2)2

  =

  4

  (k + 1)2((k + 1) + 1)2

  =

  4

  Therefore 13 + 23 + 33 + 43 + · · · + k3 + (k + 1)3 = (k+1)2((k+1)+1)2

  4

  , which means the

  statement is true for n = k + 1.

  ■

  278

  Solutions

  5. If n ∈ N, then 21 + 22 + 23 + ··· + 2n = 2n+1 − 2.

  Proof. The proof is by mathematical induction.

  (1) When n = 1, this statement is 21 = 21+1 − 2, or 2 = 4 − 2, which is true.

  (2) Now assume the statement is true for some integer n = k ≥ 1, that is assume

  21 + 22 + 23 + ··· + 2k = 2k+1 − 2. Observe this implies that the statement is true

  for n = k + 1, as follows:

  21 + 22 + 23 + ··· + 2k + 2k+1

  =

  (21 + 22 + 23 + ··· + 2k) + 2k+1

  =

  2k+1 − 2 + 2k+1

  =

  2 · 2k+1 − 2

  =

  2k+2 − 2

  =

  2(k+1)+1 − 2

  Thus we have 21 + 22 + 23 + · · · + 2k + 2k+1 = 2(k+1)+1 − 2, so the statement is true

  for n = k + 1.

  Thus the result follows by mathematical induction.

  ■

  n(n + 1)(2n + 7)

  7. If n ∈ N, then 1 · 3 + 2 · 4 + 3 · 5 + 4 · 6 + ··· + n(n + 2) =

  .

  6

  Proof. The proof is by mathematical induction.

  (1) When n = 1, we have 1 · 3 = 1(1+1)(2+7)

  6

  , which is the true statement 3 = 18

  6 .

  (2) Now assume the statement is true for some integer n = k ≥ 1, that is assume

  1 · 3 + 2 · 4 + 3 · 5 + 4 · 6 + ··· + k(k + 2) = k(k+1)(2k+7)

  6

  . Now observe that

  1 · 3 + 2 · 4 + 3 · 5 + 4 · 6 + ··· + k(k + 2) + (k + 1)((k + 1) + 2) =

  (1 · 3 + 2 · 4 + 3 · 5 + 4 · 6 + ··· + k(k + 2)) + (k + 1)((k + 1) + 2) =

  k(k + 1)(2k + 7) +(k+1)((k+1)+2) =

  6

  k(k + 1)(2k + 7)

  6(k + 1)(k + 3)

  +

  =

  6

  6

  k(k + 1)(2k + 7) + 6(k + 1)(k + 3)

  =

  6

  (k + 1)(k(2k + 7) + 6(k + 3))

  =

  6

  (k + 1)(2k2 + 13k + 18)

  =

  6

  (k + 1)(k + 2)(2k + 9)

  =

  6

  (k + 1)((k + 1) + 1)(2(k + 1) + 7)

  6

  Thus we have 1·3+2·4+3·5+4·6+· · ·+k(k+2)+(k+1)((k+1)+2) = (k+1)((k+1)+1)(2(k+1)+7)

  6

  ,

  and this means the statement is true for n = k + 1.

  Thus the result follows by mathematical induction.

  ■

  279

  9. For any integer n ≥ 0, it follows that 24 | (52n − 1).

  Proof. The proof is by mathematical induction.

  (1) For n = 0, the statement is 24 | (52·0 − 1). This is 24 | 0, which is true.

  (2) Now assume the statement is true for some integer n = k ≥ 1, that is assume

  24 | (52k − 1). This means 52k − 1 = 24a for some integer a, and from this we

  get 52k = 24a + 1. Now observe that

  52(k+1) − 1 =

  52k+2 − 1 =

  5252k − 1 =

  52(24a + 1) − 1 =

  25(24a + 1) − 1 =

  25 · 24a + 25 − 1 = 24(25a + 1).

  This shows 52(k+1) − 1 = 24(25a + 1), which means 24 | 52(k+1) − 1.

  This completes the proof by mathematical induction.

  ■

  11. For any integer n ≥ 0, it follows that 3 | (n3 + 5n + 6).

  Proof. The proof is by mathematical induction.

  (1) When n = 0, the statement is 3 | (03 + 5 · 0 + 6), or 3 | 6, which is true.

  (2) Now assume the statement is true for some integer n = k ≥ 0, that is assume

  3 | (k3 + 5k + 6). This means k3 + 5k + 6 = 3a for some integer a. We need to

  show that 3 | ((k + 1)3 + 5(k + 1) + 6). Observe that

  (k + 1)3 + 5(k + 1) + 6 =

  k3 + 3k2 + 3k + 1 + 5k + 5 + 6

  =

  (k3 + 5k + 6) + 3k2 + 3k + 6

  =

  3a + 3k2 + 3k + 6

  =

  3(a + k2 + k + 2).

  Thus we have deduced (k + 1)3 − (k + 1) = 3(a + k2 + k + 2). Since a + k2 + k + 2 is

  an integer, it follows that 3
| ((k + 1)3 + 5(k + 1) + 6).

  It follows by mathematical induction that 3 | (n3 + 5n + 6) for every n ≥ 0.

  ■

  13. For any integer n ≥ 0, it follows that 6 | (n3 − n).

  Proof. The proof is by mathematical induction.

  (1) When n = 0, the statement is 6 | (03 − 0), or 6 | 0, which is true.

  280

  Solutions

  (2) Now assume the statement is true for some integer n = k ≥ 0, that is, assume

  6 | (k3 − k). This means k3 − k = 6a for some integer a. We need to show that

  6 | ((k + 1)3 − (k + 1)). Observe that

  (k + 1)3 − (k + 1) =

  k3 + 3k2 + 3k + 1 − k − 1

  =

  (k3 − k) + 3k2 + 3k

  =

  6a + 3k2 + 3k

  =

  6a + 3k(k + 1).

  Thus we have deduced (k + 1)3 − (k + 1) = 6a + 3k(k + 1). Since one of k or (k + 1)

  must be even, it follows that k(k + 1) is even, so k(k + 1) = 2b for some integer

  b. Consequently (k + 1)3 − (k + 1) = 6a + 3k(k + 1) = 6a + 3(2b) = 6(a + b). Since

  (k + 1)3 − (k + 1) = 6(a + b) it follows that 6 | ((k + 1)3 − (k + 1)).

  Thus the result follows by mathematical induction.

  ■

  15.

  1

  If n ∈ N, then 1·2 + 1

  2·3 + 1

  3·4 + 1

  4·5 + · · · +

  1

  n(n+1) = 1 − 1

  n+1 .

  Proof. The proof is by mathematical induction.

  1

  1

  (1) When n = 1, the statement is 1(1+1) = 1 − 1

  1+1 , which simplifies to 2 = 1

  2 .

  (2) Now assume the statement is true for some integer n = k ≥ 1, that is assume

  1

  1·2 + 1

  2·3 + 1

  3·4 + 1

  4·5 + · · · +

  1

  k(k+1) = 1 − 1

  k+1 . Next we show that the statement for

  n = k + 1 is true. Observe that

  1

  1

  1

  1

  1

  1

  +

  +

  +

  + · · · +

  +

  =

  1 · 2

  2 · 3

  3 · 4

  4 · 5

  k(k + 1)

  (k + 1)((k + 1) + 1)

  µ

  1

  1

  1

  1

  1

  ¶

  1

  +

  +

  +

  + · · · +

  +

  =

  1 · 2

  2 · 3

  3 · 4

  4 · 5

  k(k + 1)

  (k + 1)(k + 2)

  µ

  1 ¶

  1

  1 −

  +

  =

  k + 1

  (k + 1)(k + 2)

  1

  1

  1 −

  +

  =

  k + 1

  (k + 1)(k + 2)

  k + 2

  1

  1 −

  +

  =

  (k + 1)(k + 2)

  (k + 1)(k + 2)

  k + 1

  1 −

  =

  (k + 1)(k + 2)

  1

  1 −

  =

  k + 2

  1

  1 −

  .

  (k + 1) + 1

  1

  This establishes

  ,

  1·2 + 1

  2·3 + 1

  3·4 + 1

  4·5 + · · · +

  1

  (k+1)((k+1)+1 = 1 −

  1

  (k+1)+1 which is to say

  that the statement is true for n = k + 1.

  This completes the proof by mathematical induction.

  ■

  281

  17. Suppose A1, A2,... An are sets in some universal set U, and n ≥ 2. Prove that

  A1 ∩ A2 ∩ ··· ∩ An = A1 ∪ A2 ∪ ··· ∪ An.

  Proof. The proof is by strong induction.

  (1) When n = 2 the statement is A1 ∩ A2 = A1 ∪ A2. This is not an entirely

  obvious statement, so we have to prove it. Observe that

  A1 ∩ A2

  =

  {x : (x ∈ U) ∧ (x ∉ A1 ∩ A2)} (definition of complement)

  =

  {x : (x ∈ U)∧ ∼ (x ∈ A1 ∩ A2)}

  =

  {x : (x ∈ U)∧ ∼ ((x ∈ A1) ∧ (x ∈ A2))} (definition of ∩)

  =

  {x : (x ∈ U) ∧ (∼ (x ∈ A1)∨ ∼ (x ∈ A2))} (DeMorgan)

  =

  {x : (x ∈ U) ∧ ((x ∉ A1) ∨ (x ∉ A2))}

  =

  {x : (x ∈ U) ∧ (x ∉ A1) ∨ (x ∈ U) ∧ (x ∉ A2)} (distributive prop.)

  =

  {x : ((x ∈ U) ∧ (x ∉ A1))} ∪ {x : ((x ∈ U) ∧ (x ∉ A2))} (def. of ∪)

  =

  A1 ∪ A2 (definition of complement)

  (2) Let k ≥ 2. Assume the statement is true if it involves k or fewer sets. Then

  A1 ∩ A2 ∩ ··· ∩ Ak−1 ∩ Ak ∩ Ak+1 =

  A1 ∩ A2 ∩ ··· ∩ Ak−1 ∩ (Ak ∩ Ak+1) = A1 ∪ A2 ∪ ··· ∪ Ak−1 ∪ Ak ∩ Ak+1

  =

  A1 ∪ A2 ∪ ··· ∪ Ak−1 ∪ Ak ∪ Ak+1

  Thus the statement is true when it involves k + 1 sets.

  This completes the proof by strong induction.

  ■

  19.

  Pn

  Prove

  1/k2 ≤ 2 − 1/n

  k=1

  for every n.

  Proof. This clearly holds for n = 1. Assume it holds for some n ≥ 1. Then

  Pn+1 1/k2 ≤ 2−1/n+1/(n+1)2 = 2− (n+1)2−n

  k=1

  n(n+1)2 ≤ 2 − 1/(n + 1). The proof is complete.

  ■

  21.

  1

  If n ∈ N, then 1 + 12 + 13 + · · · + 1

  2n ≥ 1 + n

  2 .

  Proof. If n = 1, the result is obvious.

  Assume the proposition holds for some n > 1. Then

  1

  1

  1

  1

  µ 1

  1

  1

  1 ¶

  µ

  1

  1

  1

  1 ¶

  +

  +

  + · · · +

  =

  +

  +

  + · · · +

  +

  +

  +

  + · · · +

  1

  2

  3

  2n+1

  1

  2

  3

  2n

  2n + 1

  2n + 2

  2n + 3

  2n+1

  µ

  ¶

  ³

  n ´

  1

  1

  1

  1

  ≥

  1 +

  +

  +

  +

  + · · · +

  .

  2

  2n + 1

  2n + 2

  2n + 3

  2n+1

  ³

  1

  Ńow, the sum 2n+1 + 1

  2n+2 +

  1

  2n+3 + · · · +

  1

  2n+1

  on the right has 2n+1 −2n = 2n terms,

  1

  all greater than or equal to 2n+1 , so the sum is greater than 2n 1

  2n+1 = 1

  2 . Therefore

  1

  ³

  ´

  ¢

  1<
br />
  ¢

  we get 1 + 12 + 13 + · · · + 1

  +

  ≥ ¡1 + n + 1

  2n+1 ≥ ¡1 + n

  2

  2n+1 +

  1

  2n+2 +

  1

  2n+3 + · · · +

  1

  2n+1

  2

  2 =

  1 + n+1

  2 . This means the result is true for n + 1, so the theorem is proved.

  ■

  282

  Solutions

  23.

  ¡n¢

  Use induction to prove the binomial theorem (x + y)n = Pn

  xn−i yi

  i=0 i

  .

  Proof.

  ¢

  ¢

  Notice that when n = 1, the formula is (x + y)1 = ¡1 x1 y0

  x0 y1

  0

  + ¡11

  = x + y,

  which is true.

  Now assume the theorem is true for some n > 1. We will show that this implies

  that it is true for the power n + 1. Just observe that

  (x + y)n+1

  =

  (x + y)(x + y)n

  n à !

  n

  X

  =

  (x + y)

  xn−i yi

  i

  i=0

  n à !

  Ã

  !

  n

  n

  n

  X

  X

  =

  x(n+1)−i yi +

  xn−i yi+1

  i

  i

  i=0

  i=0

  n "Ã !

  Ã

  !#

  n

  n

  X

  =

  +

  x(n+1)−i yi

  + yn+1

  i

  i

  i=0

  − 1

  n Ã

  !

  Ã

  !

  n

  n

  X

  + 1

  + 1

  =

  x(n+1)−i yi

  +

  yn+1

  i

  n

  i=0

  + 1

  n+1 Ã

  !

  n

  X

  + 1

  =

  x(n+1)−i yi.

  i

  i=0

  This shows that the formula is true for (x + y)n+1, so the theorem is proved.

  ■

  25. Concerning the Fibonacci sequence, prove that F1+F2+F3+F4+...+Fn = Fn+2−1.

  Proof. The proof is by induction.

  (1) When n = 1 the statement is F1 = F1+2 − 1 = F3 − 1 = 2 − 1 = 1, which is true.

  Also when n = 2 the statement is F1 + F2 = F2+2 − 1 = F4 − 1 = 3 − 1 = 2, which is

  true, as F1 + F2 = 1 + 1 = 2.

  (2) Now assume k ≥ 1 and F1 + F2 + F3 + F4 + . . . + Fk = Fk+2 − 1. We need to show

  F1 + F2 + F3 + F4 + ... + Fk + Fk+1 = Fk+3 − 1. Observe that

  F1 + F2 + F3 + F4 + ... + Fk + Fk+1 =

  (F1 + F2 + F3 + F4 + ... + Fk) + Fk+1 =

  Fk+2 − 1 + +Fk+1 = (Fk+1 + Fk+2) − 1

  =

  Fk+3 − 1.

  This completes the proof by induction.

  ■

  27. Concerning the Fibonacci sequence, prove that F1 + F3 + ··· + F2n−1 = F2n.

  Proof. If n = 1, the result is immediate. Assume for some n > 1 we have

  Pn

  F

  Pn+1 F

  F

  i=1

  2i−1 = F2n. Then

  i=1

  2i−1 = F2n+1+Pn

 

‹ Prev